Định lý Muirhead và ứng dụng

Cách chứng minh định lý Muirhead cho n số tương đối phức tạp. Và hơn nữa trong thực tế ta cũng không mấy khi cần sử dụng đến trường hợp tổng quát này. Do đó trong khuôn khổ bài báo này tôi xin đi sâu vào trường hợp thông dụng nhất của định lý. Tôi sẽ chứng minh và đưa ra các ứng dụng của định lý trong trường hợp .

doc5 trang | Chia sẻ: thanhthanh29 | Lượt xem: 612 | Lượt tải: 0download
Bạn đang xem nội dung tài liệu Định lý Muirhead và ứng dụng, để tải tài liệu về máy bạn click vào nút DOWNLOAD ở trên
ĐỊNH LÝ MUIRHEAD VÀ ỨNG DỤNG Trong bài viết này tôi xin giới thiệu về định lý Muirhead và ứng dụng của nó trong các bài toán chứng minh bất đẳng thức. Trước hết ta phát biểu định lý Muirhead dạng tổng quát cho n số: Cho n số dương . Ký hiệu là tổng đối xứng của các biểu thức dạng . Ta nói bộ số trội hơn bộ số và ký hiệu: nếu: Khi đó: Đẳng thức xảy ra khi và chỉ khi hoặc . Cách chứng minh định lý Muirhead cho n số tương đối phức tạp. Và hơn nữa trong thực tế ta cũng không mấy khi cần sử dụng đến trường hợp tổng quát này. Do đó trong khuôn khổ bài báo này tôi xin đi sâu vào trường hợp thông dụng nhất của định lý. Tôi sẽ chứng minh và đưa ra các ứng dụng của định lý trong trường hợp . Định lý Muirhead cho 3 số: Cho các số thực thoả mãn: Cho x, y, z là các số thực dương. Khi đó: . Đẳng thức xảy ra khi và chỉ khi hoặc . Chứng minh: Bổ đề: Cho các số dương thoả mãn và . Khi đó với các số x, y dương ta có: . CM: Không mất tổng quát ta giả sử . Dễ dàng có: . Áp dụng: Ta xét 2 trường hợp: a) Và Áp dụng bổ đề hai lần ta được: ( là kí hiệu tổng hoán vị) b) Áp dụng bổ đề hai lần ta được: Định lý được chứng minh. Bây giờ tôi sẽ đưa ra các ví dụ minh hoạ để các bạn thấy được ứng dụng của định lý trong các bài toán cụ thể. Ví dụ 1: CMR: và ta có: CM: Bất đẳng thức cần chứng minh tương đương với: Do nên Bất đẳng thức cuối cùng đúng theo Muirhead. Đẳng thức xảy ra khi và chỉ khi . Nhận xét: Bài toán trên nếu ta chọn và đặt thì ta sẽ thu đuợc bài toán IMO 1995. Ví dụ 2: Cho và . CMR: CM: Bất đẳng thức cần chứng minh tương đương với: Theo BĐT Cauchy-Schwarz để chứng minh bài toán rõ ràng ta chỉ cần chứng minh khẳng định sau: Bất đẳng thức cuối cùng đúng theo Muirhead. Đẳng thức xảy ra khi và chỉ khi . Ví dụ 3: (USAMO 1997) CMR: ta có: CM: Bất đẳng thức cần chứng minh tương đương với: Bất đẳng thức cuối cùng đúng theo Muirhead. Đẳng thức xảy ra khi và chỉ khi . Ví dụ 4: Cho . CMR: CM: Vì nên để chứng minh bài toán ta sẽ chứng minh khẳng định sau: Bất đẳng thức cuối cùng đúng theo Muirhead. Đẳng thức xảy ra khi và chỉ khi . Qua các ví dụ trên có thể thấy rằng để sử dụng hiệu quả định lý Muirhead trong chứng minh thì chúng ta phải tìm cách đưa bài toán bất đẳng thức về dạng đối xứng, đồng bậc. Ngoài ra các bạn cũng có thể nhận thấy rằng các bài toán giải bằng định lý Muirhead thường không khó về đường lối mà chỉ đòi hỏi kỹ năng biến đổi toán học chính xác. Bên cạnh đó cũng có bạn có thể nhận xét rằng định lý Muirhead chỉ hiệu quả đối với lớp bài toán mà dấu đẳng thức xẩy ra khi tất cả các biến bằng nhau. Điều này thực ra cũng có một ngoại lệ. Ta hãy xét trường hợp các biến x, y, z có thể bằng 0 và tất nhiên điều kiện đi kèm phải là . Khi đó đẳng thức trong định lý Muirhead sẽ xẩy ra khi và chỉ khi hoặc hoặc trong 3 số x, y, z có 2 số bằng nhau, số còn lại bằng 0. Ví dụ sau minh hoạ cho trường hợp này: Ví dụ 5: Cho thoả mãn . CMR: CM: Bất đẳng thức cần chứng minh tương đương với: Rút gọn BĐT này ta được BĐT tương đương sau đây: Bất đẳng thức cuối cùng đúng theo Muirhead. Đẳng thức xảy ra khi và chỉ khi : Thông qua bài viết này tôi hy vọng rằng mỗi bạn đều đã rút ra được những nhận xét và kinh nghiệm cho riêng mình. Để kết thúc xin gửi tới các bạn một số bài tập tương tự sau để các bạn rèn luyện thêm: Bài 1: Cho thoả mãn . CMR: Bài 2: (Iran 1996) Cho . CMR: Bài 3: Cho là độ dài 3 cạnh tam giác. CMR:

File đính kèm:

  • docDINH LY MUIRHEAD VA UNG DUNG.doc
Giáo án liên quan